Replace :=/=: with \coloneq/\eqcolon

This commit is contained in:
Anton Mosich 2023-11-14 12:37:13 +01:00
parent ff2b11213d
commit f342350b51
No known key found for this signature in database
GPG Key ID: 28F77AAFFAF5BAF6
1 changed files with 134 additions and 133 deletions

View File

@ -157,10 +157,10 @@ postaction={decorate}},
\section{Permutationen}
\begin{defin}
Sei $n \in \mathbb{N} \setminus \{0\}, [n] := \{1, 2, \dots, n\}$. \\
Sei $n \in \mathbb{N} \setminus \{0\}, [n] \coloneq \{1, 2, \dots, n\}$. \\
Eine bijektive Abbildung $\pi\colon[n]\to[n]$ heißt \underline{Permutation} von $[n]$.
Wir definieren die \underline{symmetrische Gruppe}
$S_n := \{\pi\text{ Permutation von }[n]\}$
$S_n \coloneq \{\pi\text{ Permutation von }[n]\}$
mit der Hintereinanderausführung als Gruppenoperation.
\end{defin}
@ -231,7 +231,7 @@ postaction={decorate}},
(3 1 2) = (2 1)(3 1) = (3 1)(3 2)
\]
\item $f\in \mathbb{Z}[X_1, \dots, X_n], \pi \in S_n$ \\
$\pi f(X_1, \dots, X_n) := f(X_{\pi(1)}, \dots, X_{\pi(n)})$
$\pi f(X_1, \dots, X_n) \coloneq f(X_{\pi(1)}, \dots, X_{\pi(n)})$
\end{itemize}
\subsubsection{Beispiel}
$\pi = (2 3 1), f(X_1, X_2, X_3) = X_1-X_2+X_1X_3 \implies \pi f(X_1, X_2, X_3) = X_2 - X_3 + X_2X_1$
@ -255,8 +255,8 @@ $\pi = (2 3 1), f(X_1, X_2, X_3) = X_1-X_2+X_1X_3 \implies \pi f(X_1, X_2, X_3)
\begin{align*}
\pi f(X_1, \dots, X_n) & = \prod_{i<j}(X_{\pi(j)}-X_{\pi(i)}) \\
& =\Bigl(\prod_{\substack{i<j \\
\pi(i)<\pi(j)}}
(X_{\pi(j)}-X_{\pi(i)})\Bigr)
\pi(i)<\pi(j)}}
(X_{\pi(j)}-X_{\pi(i)})\Bigr)
\Bigl(\prod_{\substack{i<j \\
\pi(j)<\pi(i)}}(X_{\pi(j)}-X_{\pi(i)})\Bigr) \\
& = (-1)^{\abs{\{(i, j)\in[n]\times[n]\colon i<j\land\pi(i)>\pi(j)\}}}
@ -409,7 +409,7 @@ $\varphi$ alternierend und $a_i = a_j$ für $i\neq j \implies \varphi(a_1, \dots
\lambda_{1j_1}\cdots\lambda_{nj_n}} \\
& \underbrace{=}_{\mathclap{\varphi\text{ alternierend}}}
\sum_{\substack{j_1, \dots, j_n \\
\text{paarweise verschieden}}}
\text{paarweise verschieden}}}
{\varphi(b_{j_1}, \dots, b_{j_n})\lambda_{1j_1} \cdots \lambda_{nj_n}} \\
& = \sum_{\pi\in S_n} \varphi(b_{\pi(1)}, \dots, b_{\pi(n)})
\lambda_{1\pi(1)} \cdots \lambda_{n\pi(n)} \\
@ -482,7 +482,7 @@ alternierende n-Linearform.
\begin{proof}
Sei $a_1, \dots, a_n$ Basis von V. Nach Lemma \ref{theo:1.2.4} ist
$\varphi_i(a_1, \dots, a_n)\neq0, i=1, 2$.\\
Sei $c:=\dfrac{\varphi_1(a_1, \dots, a_n)}{\varphi_2(a_1, \dots, a_n)} \in \K\setminus\{0\}$.\\
Sei $c\coloneq\dfrac{\varphi_1(a_1, \dots, a_n)}{\varphi_2(a_1, \dots, a_n)} \in \K\setminus\{0\}$.\\
Sei $b_1, \dots, b_n$ mit $b_i=\sum\lambda_{ij}a_j$.\\
Dann gilt nach Satz \ref{theo:1.2.5}(a), dass für $i=1, 2$
\begin{align*}
@ -499,8 +499,8 @@ alternierende n-Linearform.
Sei $B=(a_1, \dots, a_n)$ Basis des \K-Vektorraums V.
Sei $\varphi$ nicht ausgeartete n-Linearform und $\alpha \in \homkv$.
Dann ist die \underline{Determinante von $\alpha$} definiert durch \[
\det(\alpha):=\det{}_\K(\alpha)
:=\frac{\varphi(\alpha(a_1), \dots, \alpha(a_n))}{\varphi(a_1, \dots, a_n)}
\det(\alpha)\coloneq\det{}_\K(\alpha)
\coloneq\frac{\varphi(\alpha(a_1), \dots, \alpha(a_n))}{\varphi(a_1, \dots, a_n)}
\]
\end{defin}
@ -519,7 +519,7 @@ alternierende n-Linearform.
ausgeartet,
\[\varphi(\alpha(a_1), \dots, \alpha(a_n))\neq0\]
Sei $\varphi_\alpha(b_1, \dots, b_n) := \varphi(\alpha(b_1), \dots,
Sei $\varphi_\alpha(b_1, \dots, b_n) \coloneq \varphi(\alpha(b_1), \dots,
\alpha(b_n))$. Dann ist $\varphi_\alpha$ alternierend und nicht ausgeartet.
Wegen Satz \ref{theo:1.2.6} folgt, dass $c\in\K\setminus\{0\}$ existiert mit
\begin{equation}
@ -530,9 +530,10 @@ alternierende n-Linearform.
Da \ref{eq:constantphi} unabhängig von B ist also $\det(\alpha)$ unabhängig von B.
Sei nun $\psi$ eine zweite alternierende, nicht ausgeartete n-Form und
$\psi_\alpha(b_1, \dots, b_n) := \psi(\alpha(b_1), \dots, \alpha(b_n))$. Dann
ist $\psi_\alpha$ alternierend und nicht ausgeartet. Nach Satz \ref{theo:1.2.6}
gibt es $d\in\K\setminus\{0\} \text{ mit }d=\frac\psi\varphi$. Also gilt:
$\psi_\alpha(b_1, \dots, b_n) \coloneq \psi(\alpha(b_1), \dots, \alpha(b_n))$.
Dann ist $\psi_\alpha$ alternierend und nicht ausgeartet. Nach Satz
\ref{theo:1.2.6} gibt es $d\in\K\setminus\{0\} \text{ mit }d=\frac\psi\varphi$.
Also gilt:
\[
\det(\alpha)=\frac{\varphi_\alpha(a_1, \dots, a_n)}{\varphi(a_1, \dots, a_n)}=
\frac{d\varphi_\alpha(a_1, \dots, a_n)}{d\varphi(a_1, \dots, a_n)}=
@ -651,7 +652,7 @@ Schreibweise für $A=(a_{ij})$:
\det(A^T) & = \sum_{\pi\in S_n}\sgn(\pi)a_{\pi(1)1}\cdots a_{\pi(n)n} \\
& =\sum_{\pi\in S_n}\sgn(\pi)a_{1\pi^{-1}(1)}\cdots a_{n\pi^{-1}(n)} \\
& \underbrace{=}_{\substack{\sgn(\pi^{-1})=\sgn(\pi) \\
\pi^{-1}\mapsto\pi}} \sum_{\pi\in S_n} \sgn(\pi)a_{1\pi(1)}\cdots a_{n\pi(n)}
\pi^{-1}\mapsto\pi}} \sum_{\pi\in S_n} \sgn(\pi)a_{1\pi(1)}\cdots a_{n\pi(n)}
\end{aligned}
\end{equation*}
\item[b) - i)] folgt daraus, dass für \[\alpha\colon
@ -741,7 +742,7 @@ $n>3 \to $ Gaußalgorithmus
\begin{defin}
Sei $A\in\K^{n\times n}$ und $i, j\in[n]$. Sei $M_{ij}\in\K^{n\times n}$ die Matrix,
welche durch Ersetzung der j-ten Spalte durch den i-ten Einheitsvektor $e_j$ entsteht.\\
$A_{ij}:=\det(M_{ij})$ heißt \underline{Kofaktor} (zum Indexpaar $(i, j)$).
$A_{ij}\coloneq\det(M_{ij})$ heißt \underline{Kofaktor} (zum Indexpaar $(i, j)$).
\begin{equation*}
\bordermatrix{
&&&&j \cr
@ -774,7 +775,7 @@ da obige Matrix aus $M_{ij}$ durch Spaltenadditionen hervorgeht.
Sei $\tilde{A_{ij}}\in\K^{(n-1)\times(n-1)}$ die Matrix, welche aus A durch
Streichung der i-ten Spalte und j-ten Zeile hervorgeht und
$D_{ij}:=\det(\tilde{A_{ij}})$. Dann gilt \[A_{ij}=(-1)^{i+j}D_{ij}\]
$D_{ij}\coloneq\det(\tilde{A_{ij}})$. Dann gilt \[A_{ij}=(-1)^{i+j}D_{ij}\]
\end{lemma}
\begin{proof}
Transformiere durch ($i-1$) Spaltenvertauschungen und ($j-1$) Zeilenvertauschungen die Matrix
@ -916,7 +917,7 @@ da obige Matrix aus $M_{ij}$ durch Spaltenadditionen hervorgeht.
$D\in \K^{n\times n}$ heißt \underline{Diagonalmatrix} wenn $\forall i\neq j\colon d_{ij}=0$.
Wir schreiben auch
\[
\diag(\lambda_1, \dots, \lambda_n):=
\diag(\lambda_1, \dots, \lambda_n)\coloneq
\begin{pmatrix}
\lambda_1 & 0 & \dots & 0 \\
0 & \lambda_2 & \dots & 0 \\
@ -1016,10 +1017,10 @@ da obige Matrix aus $M_{ij}$ durch Spaltenadditionen hervorgeht.
\begin{defin}
\begin{enumerate}[label=\alph*)]
\item Sei $\alpha \in \homkv$ und $\lambda \in \spec(\alpha)$. Dann heißt
$\eig_\alpha(\lambda):=\{v\in V\colon \alpha(v) = \lambda v \}$ der zugehörige
\underline{Eigenraum}.
$\eig_\alpha(\lambda)\coloneq\{v\in V\colon \alpha(v) = \lambda v \}$ der
zugehörige \underline{Eigenraum}.
\item Sei $A \in \K^{n\times n}$ und $\lambda \in \spec(A)$. Dann heißt
$\eig_A(\lambda):=\{v\in \K^n\colon A v = \lambda v \}$ der zugehörige
$\eig_A(\lambda)\coloneq\{v\in \K^n\colon A v = \lambda v \}$ der zugehörige
\underline{Eigenraum}.
\end{enumerate}
\end{defin}
@ -1167,15 +1168,15 @@ $\le\genfrac{}{}{0pt}{0}{\dim(V)}{n}$, da
\right)} \\
& \left(
\begin{array}{c c | c}
\bar1 & \bar4 & \bar0 \\
\bar1 & \bar4 & \bar0
\end{array}
\bar1 & \bar4 & \bar0 \\
\bar1 & \bar4 & \bar0
\end{array}
\right) \\
& \left(
\begin{array}{c c | c}
\bar1 & \bar4 & \bar0 \\
\bar0 & \bar0 & \bar0
\end{array}
\bar1 & \bar4 & \bar0 \\
\bar0 & \bar0 & \bar0
\end{array}
\right) \\
& \implies \eig_{\bar2}(A) = \linspan{
\begin{pmatrix}
@ -1276,7 +1277,7 @@ $\le\genfrac{}{}{0pt}{0}{\dim(V)}{n}$, da
\end{satz}
\begin{proof}
Klarerweise gilt $1\le m_g(\mu)$ da $\mu$ Eigenwert ist.
Sei $r:= m_g(\mu)$ und $b_1, \dots, b_r$ Basis von $\eig_\alpha(\mu)$. Sei $B=(b_1, \dots, b_n)$ Basis.
Sei $r\coloneq m_g(\mu)$ und $b_1, \dots, b_r$ Basis von $\eig_\alpha(\mu)$. Sei $B=(b_1, \dots, b_n)$ Basis.
Dann ist
\[ {}_B M(\alpha)_B =
\bordermatrix{
@ -1350,7 +1351,7 @@ $\le\genfrac{}{}{0pt}{0}{\dim(V)}{n}$, da
\item[$\impliedby$:] Aus i), ii) folgt, dass
\begin{equation}
\label{eq:2.2.15.1}
\sum_{i=1}^r \underbrace{\dim(\eig_\alpha(\lambda_i))}_{=m_g(\lambda_i)=:d_i} = n
\sum_{i=1}^r \underbrace{\dim(\eig_\alpha(\lambda_i))}_{=m_g(\lambda_i)\eqcolon d_i} = n
\end{equation}
Sei $(b_i^1, \dots, b_i^{d_i})$ Basis von $\eig_\alpha(\lambda_i)$.
Wir zeigen, dass $B=\{b_i^1, \dots, b_i^{d_i}\colon i=1, \dots, r\}$ Basis ist.
@ -1417,7 +1418,7 @@ $
\end{vmatrix}
\\
\underbrace{=}_{\mathclap{\substack{\text{Entwicklung} \\
\text{nach 1. Zeile}}}}
\text{nach 1. Zeile}}}}
& (1-\lambda)
\begin{vmatrix}
-2 -\lambda & 1 \\
@ -1444,31 +1445,31 @@ $
\begin{array}{c c c | c} 1-3 & 2 & 2 & 0 \\
2 & -2-3 & 1 & 0 \\
2 & 1 & -2-3 & 0
\end{array}
\end{array}
\right)
= \left(
\begin{array}{c c c | c} -2 & 2 & 2 & 0 \\
2 & -5 & 1 & 0 \\
2 & 1 & -5 & 0
\end{array}
\end{array}
\right) \\
& \sim \left(
\begin{array}{c c c | c} -1 & 1 & 1 & 0 \\
0 & -3 & 3 & 0 \\
0 & 3 & -3 & 0
\end{array}
\end{array}
\right)
\sim \left(
\begin{array}{c c c | c} 1 & -1 & -1 & 0 \\
0 & 1 & -1 & 0 \\
0 & 0 & 0 & 0
\end{array}
\end{array}
\right)
\sim \left(
\begin{array}{c c c | c} 1 & 0 & -2 & 0 \\
0 & 1 & -1 & 0 \\
0 & 0 & 0 & 0
\end{array}
\end{array}
\right) \\
& \implies \eig_A(3) = \linspan{
\begin{pmatrix}
@ -1485,19 +1486,19 @@ $
\begin{array}{c c c | c} 1+3 & 2 & 2 & 0 \\
2 & -2+3 & 1 & 0 \\
2 & 1 & -2+3 & 0
\end{array}
\end{array}
\right)
= \left(
\begin{array}{c c c | c} 4 & 2 & 2 & 0 \\
2 & 1 & 1 & 0 \\
2 & 1 & 1 & 0
\end{array}
\end{array}
\right) \\
& \sim \left(
\begin{array}{c c c | c} 2 & 1 & 1 & 0 \\
0 & 0 & 0 & 0 \\
0 & 0 & 0 & 0
\end{array}
\end{array}
\right) \\
& \implies \eig_A(-3) = \linspan {
\begin{pmatrix}
@ -1536,7 +1537,7 @@ $
\begin{lemma}
\label{theo:2.2.16}
Sei $A\in\K^{\nxn}$ und $\underbrace{\spur(A)}_{\mathclap{\color{red}\text{\dq Spur von $A$ \dq}}}
:= \sum\limits_{i=1}^n a_{ii}$
\coloneq \sum\limits_{i=1}^n a_{ii}$
\[\chi_A(\lambda) = (-1)^n\lambda^n + (-1)^{n-1} \spur(A) \lambda^{n-1} + \cdots + \det(A)\]
\end{lemma}
\begin{proof}
@ -1576,7 +1577,7 @@ $
$\chi_A(\lambda)=c_n \lambda^n + c_{n-1} \lambda^{n-1} + \cdots + c_0$.
Dann gilt
\[
\chi_A(A):=c_n A^n + c_{n-1} A ^{n-1} + \cdots c_0 I = 0 =
\chi_A(A)\coloneq c_n A^n + c_{n-1} A ^{n-1} + \cdots c_0 I = 0 =
\begin{pmatrix}
0 & \dots & 0 \\
\vdots & \ddots & \vdots \\
@ -1586,7 +1587,7 @@ $
\]
\end{satz}
\begin{proof}
Sei $B := A^T - \lambda I =
Sei $B \coloneq A^T - \lambda I =
\begin{pmatrix}
a_{11} - \lambda & a_{21} & \dots & a_{n1} \\
a_{12} & a_{22} - \lambda & \dots & a_{n2} \\
@ -1594,7 +1595,7 @@ $
a_{1n} & a_{2n} & \dots & a_{nn} - \lambda
\end{pmatrix}
= (a_{ji} - \delta_{ij} \lambda)_{ij}$
und $C:= \adj(B)$, sodass
und $C\coloneq \adj(B)$, sodass
\begin{equation}
CB \overset{\text{\ref{theo:1.4.7}}}{=} \det(B) I_n = \chi_A \cdot I_n \; [\chi_A = \chi_{A^T}]
\label{eq:2.2.18.1}
@ -1628,15 +1629,15 @@ Sei $f(\lambda) \underbrace{=}_{\text{(*)}} \prod\limits_{j=1}^{n}(\lambda_j -
\lambda) = \underbrace{c_n}_{=(-1)^n}\lambda^n + c_{n-1}\lambda ^{n-1} + \cdots
+ c_0$ Wie können wir $c_j$ effizient bestimmen?
\[
\sigma_j := (-1)^j \sum\limits_{\substack{S\subset [n] \\ \abs{ S } = n-j}}
\sigma_j \coloneq (-1)^j \sum\limits_{\substack{S\subset [n] \\ \abs{ S } = n-j}}
\prod\limits_{s \in S} \lambda_s
\]
\begin{itemize}
\item [Bemerkung 1:] $\displaystyle { c_j = (-1)^{j} \sum_{\substack{S\subseteq [n] \\
\abs{ S } = n-j}} \prod_{s \in S} \lambda_s =:
\abs{ S } = n-j}} \prod_{s \in S} \lambda_s \eqcolon
\sigma_{n-j}^n (\lambda_1, \dots, \lambda_n)}$ \\
Dies folgt aus (*) durch Ausmultiplizieren \\
Sei nun weiters $p_j^n(\lambda_1, \dots, \lambda_n) := \sum\limits_{i=1}^{n}\lambda_i^j$
Sei nun weiters $p_j^n(\lambda_1, \dots, \lambda_n) \coloneq \sum\limits_{i=1}^{n}\lambda_i^j$
\item [Bemerkung 2:] $\sigma_j^n, p_j^n$ sind symmetrisch, das heißt
\[
\begin{aligned}
@ -1666,7 +1667,7 @@ Sei $f(\lambda) \underbrace{=}_{\text{(*)}} \prod\limits_{j=1}^{n}(\lambda_j -
folgt $\sigma_n^n p_0^n + \sum\limits_{j=0}^n \sigma_j^n p_{n-j}^n = 0$ was mit
$p_0^n = n$ die gewünschte Aussage liefert.
\item [$k<n$:] Betrachte das (symmetrische) \[
q(\lambda_1, \dots, \lambda_n) :=
q(\lambda_1, \dots, \lambda_n) \coloneq
k \sigma_k^n + \sum_{j=0}^{k-1} \sigma_j^n p_{k-1}^n
\]
Es gilt \[q(\lambda_1, \dots, \lambda_n) =
@ -1816,7 +1817,7 @@ $\underset{\mathrlap{\text{\dq fast alle Matrizen sind diagonalisierbar\dq}}}
\begin{defin}
Eine $m\times m$ Matrix
\[J_m(\lambda) :=
\[J_m(\lambda) \coloneq
\begin{pmatrix}
\lambda & 1 & 0 & \dots & 0 \\
0 & \ddots & \ddots & \ddots & \vdots \\
@ -1934,7 +1935,7 @@ Wir wollen zeigen, dass $\alpha/A$ genau dann eine Jordan-Normalform besitzt, we
Das heißt ${}_B M(\alpha)_B$ ist blockdiagonal mit Jordanblöcken mit Eigenwerten $0$
\end{lemma}
\begin{proof}
Sei $V_i := \ker(\alpha^i)$. \\
Sei $V_i \coloneq \ker(\alpha^i)$. \\
Dies ergibt eine aufsteigende Kette von Unterräumen
\begin{equation*}
\underbrace{\{0\}}_{=V_0} \subseteq V_1 \subseteq \cdots \subseteq \underbrace{V_m}_{=V}
@ -1959,7 +1960,7 @@ Wir wollen zeigen, dass $\alpha/A$ genau dann eine Jordan-Normalform besitzt, we
& \implies \sum \mu_i c_i^m \in V_{m-1} \\
& \underbrace{\implies}_{\mathclap{\substack{(c_i^m) \text{ liegen} \\
\text{im Komplement} \\
\text{von } V_{m-1}}}}
\text{von } V_{m-1}}}}
\mu_i = 0, \forall i \implies \sum_{i} \mu_i \alpha(c_i^m) = 0
\end{align*}
\item[2)] folgt aus 3) [da $0\in V_{m-2}$]
@ -1971,10 +1972,10 @@ Wir wollen zeigen, dass $\alpha/A$ genau dann eine Jordan-Normalform besitzt, we
\overset{\linspan{ D^{m-1} }}{\linspan{ \alpha(C^m) } \oplus\linspan{C^{m-1}}}}_{V_{m-1}} \oplus
\overset{\linspan{ D^m }}{\linspan{C^m}} = V
\]
Setze $D^m := C^m$ und definiere induktiv für $D^i \subseteq V_i$ die Menge
$D^{i-1} := \alpha(D^i) \cup C^{i-1} \subseteq V_{i-1}$ sodass mit einer Basis
$B^{i-2}$ von $V_{i-2}$ die Menge $B^{i-2} \cup D^{i-1}$ Basis von $V_{i-1}$
ist, also
Setze $D^m \coloneq C^m$ und definiere induktiv für $D^i \subseteq V_i$ die
Menge $D^{i-1} \coloneq \alpha(D^i) \cup C^{i-1} \subseteq V_{i-1}$ sodass mit
einer Basis $B^{i-2}$ von $V_{i-2}$ die Menge $B^{i-2} \cup D^{i-1}$ Basis von
$V_{i-1}$ ist, also
\[
V_{i-2} \oplus \underbrace{\linspan{ \alpha(D^i) } \oplus \linspan {C^{i-1}}}_
{\linspan{ D^i }}
@ -2016,7 +2017,7 @@ Angenommen \(\alpha - \lambda \id\colon V \to V\) nilpotent. Dann besitzt
\begin{defin}
\label{theo:2.3.4}
Sei \(V\) \K-Vektorraum, \(\dim(V) < \infty, \alpha \in \homkv\) und \(\lambda \in \spec(\alpha)\).
Für \(l \in \mathbb{N}\) definiere \(V_{l, \lambda}:= \ker((\alpha - \lambda \id)^l)\)
Für \(l \in \mathbb{N}\) definiere \(V_{l, \lambda}\coloneq \ker((\alpha - \lambda \id)^l)\)
\end{defin}
\subsubsection{Bemerkung}
@ -2036,7 +2037,7 @@ Angenommen \(\alpha - \lambda \id\colon V \to V\) nilpotent. Dann besitzt
\label{theo:2.3.5}
Sei $V$ \K-Vektorraum, $\dim(V) < \infty, \alpha \in \homkv$. Für $l\in\mathbb{N}$ sei
$V_l := \ker(\alpha^l)$. Dann gilt $\alpha(V_l) \subseteq V_{l-1} \subseteq V_l$ für alle
$V_l \coloneq \ker(\alpha^l)$. Dann gilt $\alpha(V_l) \subseteq V_{l-1} \subseteq V_l$ für alle
$l\in \mathbb{N}$ und es existiert genau ein $k\in \mathbb{N}_0$ mit
\[
\{0\} = V_0 \subseteq V_1 \subseteq \cdots \subseteq V_k = V_{k+1} \text{ und } V_{l+1} =
@ -2054,7 +2055,7 @@ Angenommen \(\alpha - \lambda \id\colon V \to V\) nilpotent. Dann besitzt
\begin{defin}
Sei $V_{l, \lambda}$ wie in Definition \ref{theo:2.3.4} und $k$ wie in Lemma \ref{theo:2.3.5}
Dann heißt \[
\widetilde{\eig_\alpha(\lambda)} := V_{k, \lambda} = V_{k+1, \lambda}
\widetilde{\eig_\alpha(\lambda)} \coloneq V_{k, \lambda} = V_{k+1, \lambda}
\]
\underline{verallgemeinerter Eigenraum} oder \underline{Hauptraum} von $\alpha$ zum Eigenwert
$\lambda$. $v \in V_{l, \lambda} \setminus V_{l-1, \lambda}$ für $1 \le l \le k$ heißt
@ -2069,7 +2070,7 @@ Angenommen \(\alpha - \lambda \id\colon V \to V\) nilpotent. Dann besitzt
Zerlege
\begin{equation}
\label{eq:2.3.6.1}
V:= \widetilde{\eig_\alpha(\lambda_1)} \oplus \cdots \oplus \widetilde{\eig_\alpha(\lambda_r)}
V\coloneq \widetilde{\eig_\alpha(\lambda_1)} \oplus \cdots \oplus \widetilde{\eig_\alpha(\lambda_r)}
\end{equation}
dann besitzt ganz $\alpha\colon V\to V$ Jordan-Normalform
\item Sei $V= V_1 \oplus \cdots \oplus V_r$ und $\alpha \in \homkv$. Falls
@ -2103,7 +2104,7 @@ Angenommen \(\alpha - \lambda \id\colon V \to V\) nilpotent. Dann besitzt
zerfällt. Dann gilt
$V = \widetilde{\eig_\alpha(\lambda_1)} \oplus \cdots \oplus \widetilde{\eig_\alpha(\lambda_r)}$
und insbesondere $\alpha = \alpha_1 \oplus \cdots \oplus \alpha_r$ mit
$\alpha_i := \alpha|_{\widetilde{\eig_\alpha(\lambda_i)}} \in \homk(\widetilde{\eig_\alpha(\lambda_i)},
$\alpha_i \coloneq \alpha|_{\widetilde{\eig_\alpha(\lambda_i)}} \in \homk(\widetilde{\eig_\alpha(\lambda_i)},
\widetilde{\eig_\alpha(\lambda_i)})$
\end{satz}
\begin{proof}
@ -2125,7 +2126,7 @@ Angenommen \(\alpha - \lambda \id\colon V \to V\) nilpotent. Dann besitzt
\end{proof}
\item $\widetilde{\eig_\alpha(\lambda)} \neq V$. Sei $k$ minimal mit
$\ker(\alpha - \lambda - \id)^k = \widetilde{\eig_\alpha(\lambda)}$ [Lemma \ref{theo:2.3.5}]
Setze $V_1 := \widetilde{\eig_\alpha(\lambda)}, V_2 := \im(\alpha - \lambda \id)^k$. \\
Setze $V_1 \coloneq \widetilde{\eig_\alpha(\lambda)}, V_2 \coloneq \im(\alpha - \lambda \id)^k$. \\
\underline{Behauptung:}
\begin{enumerate}[label=(\roman*)]
\item $\alpha(V_i) \subseteq V_i, i \in \{1, 2\}$
@ -2156,7 +2157,7 @@ Angenommen \(\alpha - \lambda \id\colon V \to V\) nilpotent. Dann besitzt
\end{enumerate}
Es folgt $V = \underbrace{\widetilde{\eig(\lambda)}}_{V_1} \oplus V_2,
\dim(V_2) < n$ und \\ $\alpha = \alpha_1 \oplus \alpha_2, \alpha_i :=
\dim(V_2) < n$ und \\ $\alpha = \alpha_1 \oplus \alpha_2, \alpha_i \coloneq
\alpha|_{V_i}, i\in\{1, 2\}$. Es folgt $\chi_\alpha = \chi_{\alpha_1} \cdot
\chi_{\alpha_2}$, also zerfällt $\chi_{\alpha_2}$ in Linearfaktoren. Daher
können wir die Induktionsvorraussetzung anwenden, was das gewünschte Resultat
@ -2175,7 +2176,7 @@ Angenommen \(\alpha - \lambda \id\colon V \to V\) nilpotent. Dann besitzt
und \\
$\alpha = \alpha_1 \oplus \cdots \oplus \alpha_r$.
Da $\widetilde{\eig_\alpha(\lambda_i)} = \ker(\alpha - \lambda_i \id)^{k_i}$ ist
$\alpha_i - \lambda_i \id := \alpha|_{\widetilde{\eig_\alpha(\lambda_i)}} - \lambda
$\alpha_i - \lambda_i \id \coloneq \alpha|_{\widetilde{\eig_\alpha(\lambda_i)}} - \lambda
\id|_{\widetilde{\eig_\alpha(\lambda_i)}} $ nilpotent. Nach Lemma \ref{theo:2.3.3} gibt es eine Basis
$B_i$ von $\widetilde{\eig_\alpha(\lambda_i)}$ sodass ${}_{B_i} M(\alpha_i)_{B_i}$ Jordan-Normalform
hat. Es folgt mit $B= (B_1, \dots, B_r)$ dass \\ ${}_B M(\alpha)_B =
@ -2191,22 +2192,22 @@ Angenommen \(\alpha - \lambda \id\colon V \to V\) nilpotent. Dann besitzt
\item Berechne $\spec(\alpha) = \{ \lambda_1, \dots, \lambda_r \}$ \item
\begin{enumerate}[label=\alph*)]
\item Haupträume berechnen: Finde $k$ minimal mit \[
\ker(\alpha - \lambda \id)^{k+1} = \ker(\alpha - \lambda \id)^k =: V_\lambda
\ker(\alpha - \lambda \id)^{k+1} = \ker(\alpha - \lambda \id)^k \eqcolon V_\lambda
\]
\item Für $1 \le l \le k$ bestimme $B_l = \{ b_1^l, \dots, b_{r_l}^l\}$, sodass
$(B_1, \dots, B_l)$ Basis von $\ker(\alpha - \lambda \id)^l$.
\end{enumerate}
\item
\begin{enumerate}[label=\alph*)]
\item Setze zunächst $v_i^k = b_i^k, i = 1, \dots, r_k$. $D_k := (v_1^k, \dots,
v_{r_k}^k)$ \\ Setze $v_i^{k-1} := (\alpha - \lambda \id)(v_i^k) \in \linspan{
B_{k-1} }, i = 1, \dots, r_k$ \\ Ergänze gegebenenfalls $(v_1^{k-1}, \dots,
v_{r_k}^{k-1}, v_{r_{k+1}}^{k-1}, \dots, v_{r_{k-1}}^{k-1})=:D_{k-1}$, sodass
\\ $\linspan{ D_{k-1} } = \linspan {B_{k-1}}$
\item Führe 3a) iterativ aus. \\ Setze $v_i^{l-1} := (\alpha - \lambda \id)(v_i^l), i
= 1, \dots, r_l$ \\ Ergänze gegebenenfalls $v_1^{l-1}, \dots, v_{r_l}^{l-1},
v_{r_{l+1}}^{l-1}, \dots, v_{r_{l-1}}^{l-1} =:D_{l-1}$, sodass $\linspan{
D_{l-1} } = \linspan{B_{l-1}}$
\item Setze zunächst $v_i^k = b_i^k, i = 1, \dots, r_k$. $D_k \coloneq (v_1^k, \dots,
v_{r_k}^k)$ \\ Setze $v_i^{k-1} \coloneq (\alpha - \lambda \id)(v_i^k) \in
\linspan{ B_{k-1} }, i = 1, \dots, r_k$ \\ Ergänze gegebenenfalls $(v_1^{k-1},
\dots, v_{r_k}^{k-1}, v_{r_{k+1}}^{k-1}, \dots, v_{r_{k-1}}^{k-1})\eqcolon
D_{k-1}$, sodass \\ $\linspan{ D_{k-1} } = \linspan {B_{k-1}}$
\item Führe 3a) iterativ aus. \\ Setze $v_i^{l-1} \coloneq (\alpha - \lambda
\id)(v_i^l), i = 1, \dots, r_l$ \\ Ergänze gegebenenfalls $v_1^{l-1}, \dots,
v_{r_l}^{l-1}, v_{r_{l+1}}^{l-1}, \dots, v_{r_{l-1}}^{l-1} \eqcolon D_{l-1}$,
sodass $\linspan{ D_{l-1} } = \linspan{B_{l-1}}$
\end{enumerate}
\item Sei $B_\lambda = (D_1, \dots, D_k) \implies {}_{B_\lambda}
M(\alpha|_{v_\lambda})_{B_\lambda}$ hat Jordan-Normalform mit Eigenwert
@ -2290,14 +2291,14 @@ Angenommen \(\alpha - \lambda \id\colon V \to V\) nilpotent. Dann besitzt
\begin{align*}
(\overset{\mathrlap{\rotatebox{30}{\scriptsize$\in\ker(A-I)$}}}{v_1^1}
\underset{\mathclap{\substack{\rotatebox{180}{$\curvearrowright$} \\
A-I}}}{,}
A-I}}}{,}
v_1^2
\underset{\mathclap{\substack{\rotatebox{180}{$\curvearrowright$} \\
A-I}}}{,}
A-I}}}{,}
v_1^3,
\overset{\mathrlap{\rotatebox{30}{\scriptsize$\in\ker(A-I)$}}}{v_1^1}
\underset{\mathclap{\substack{\rotatebox{180}{$\curvearrowright$} \\
A-I}}}{,}
A-I}}}{,}
v_2^2) = B, {}_B M(A)_B =
\begin{pmatrix}
1 & 1 & 0 & 0 & 0 \\
@ -2488,10 +2489,10 @@ eingebettet werden kann. \newpage
\begin{defin}
Sei $V$ ein \R-Vektorraum.
\begin{align*}
& V_\C := \{ (u, v)\colon u, v\in V \} \text{
[Schreibe $(u, v) =: u + \overset{\mathclap{\substack{i^2 = -1 \\
& V_\C \coloneq \{ (u, v)\colon u, v\in V \} \text{
[Schreibe $(u, v) \eqcolon u + \overset{\mathclap{\substack{i^2 = -1 \\
|}}}{i} \cdot v$]} \\
& (u_1, v_1) + (u_2, v_2) := (u_1 + u_2, v_1 + v_2) \text{ Addition} \\
& (u_1, v_1) + (u_2, v_2) \coloneq (u_1 + u_2, v_1 + v_2) \text{ Addition} \\
& \lambda = (\gamma + i \delta) \in \C, \lambda \cdot (u, v) = (\gamma u - \delta v, \delta u + \gamma v)
\text{ skalare Multiplikation} \\
& \lambda(u + iv) = (\gamma + i \delta) (u + iv) = \gamma u + i \gamma v + i \delta u - \delta v \\
@ -2550,7 +2551,7 @@ Auch skalare Produkte können eindeutig fortgesetzt werden.
\beta_\C(u_1 + i v_1, u_2 + i v_2) & = \beta_\C(u_1, u_2 + i v_2) + i \beta_\C(v_1, u_2+iv_2) \\
& = \beta_\C(u_1, u_2) + i \beta_\C(v_1, u_2)
\underset{\mathclap{\substack{| \\
\text{\ref{theo:3.1.4} b)}}}}{-}
\text{\ref{theo:3.1.4} b)}}}}{-}
i \beta_\C(u_1, v_2) + \beta_\C(v_1, v_2) \\
& = \beta(u_1, u_2) + \beta(v_1, v_2) + i(\beta(v_1, u_2) - \beta(u_1, v_2))
\end{align*}
@ -2575,7 +2576,7 @@ Auch skalare Produkte können eindeutig fortgesetzt werden.
& = \inner uu - \overline{\lambda} \inner uv - \lambda \inner vu +
\underbrace{\lambda \overline{\lambda}}_{=\abs{\lambda}^2} \inner vv
\end{align*}
Sei $\lambda := \dfrac{\inner uv}{\inner vv}, \overline{\lambda} =
Sei $\lambda \coloneq \dfrac{\inner uv}{\inner vv}, \overline{\lambda} =
\dfrac{\overline{\inner uv }}{\overline{\inner vv }}
=\dfrac{\inner vu}{\inner vv}$, so folgt
\begin{align*}
@ -2593,9 +2594,9 @@ Auch skalare Produkte können eindeutig fortgesetzt werden.
\begin{defin}
Man nennt
\begin{itemize}
\item $\norm v := \sqrt{\inner vv }$ die \underline{Länge} oder die \underline{Norm} von
\item $\norm v \coloneq \sqrt{\inner vv }$ die \underline{Länge} oder die \underline{Norm} von
$v \in V$.
\item $\cos(\sphericalangle v w) := \dfrac{\inner vw }{\norm v \norm w }$ der
\item $\cos(\sphericalangle v w) \coloneq \dfrac{\inner vw }{\norm v \norm w }$ der
Kosinus des \underline{Winkels} zwischen $v, w \in V$. \\
(Wegen Satz \ref{theo:3.1.10} ist $\cos(\sphericalangle v w) \le 1$ und damit auch $\sphericalangle v w$
wohldefiniert!)
@ -2695,7 +2696,7 @@ Auch skalare Produkte können eindeutig fortgesetzt werden.
Sei $(a_1, a_2, \dots) \subseteq V$ linear unabhängig. Dann existiert genau ein Orthonormalsystem
$(b_1, b_2, \dots)$ mit
\begin{enumerate}[label=\roman*)]
\item $\forall k\colon \linspan{ a_1, \dots, a_k } = \linspan{b_1, \dots, b_k} =: U_k$
\item $\forall k\colon \linspan{ a_1, \dots, a_k } = \linspan{b_1, \dots, b_k} \eqcolon U_k$
\item Die Basistransformationsmatrix $M_k$ zwischen der Basen $(a_1, \dots, a_k)$ und
$(b_1, \dots, b_k)$ von $U_k$ hat positive Determinante.
\end{enumerate}
@ -2712,7 +2713,7 @@ Auch skalare Produkte können eindeutig fortgesetzt werden.
= \norm{c \cdot a_1} = \abs{ c } \norm{a_1}$ \\
$ \implies \abs{ c } = \dfrac{1}{\norm{a_1}} \implies \tilde M_k =(c)$
\item $(b_1, \dots, b_n)$ schon konstruiert mit i), ii) \\
Sei $c_{n+1} := a_{n+1} - \sum\limits_{j=1}^n \inner{a_{n+1}}{b_j} b_j$
Sei $c_{n+1} \coloneq a_{n+1} - \sum\limits_{j=1}^n \inner{a_{n+1}}{b_j} b_j$
\begin{align*}
& \forall i \in [n]\colon \inner{c_{n+1}}{b_i} = \inner{a_{n+1}}{b_i} -
\sum\limits_{j=1}^n \inner{a_{n+1}}{b_j} \underbrace{\inner{b_j}{b_i}}_{\delta_{ij}} \\
@ -2765,9 +2766,9 @@ Auch skalare Produkte können eindeutig fortgesetzt werden.
\draw [->] (0, 0) --node[above]{$a_1$} (\a1, \a2);
\draw [->] (0, 0) --node[above]{$a_2$} (\a3, \a4);
\draw [->, blue, thick] (0, 0) --node[below right]{$\inner{a_2}{b_1}b_1$} (\t1, \t2);
\draw [->, violet, very thick] (0, 0) --node[below right]{$\frac{a_1}{\norm{a_1}}=:b_1$} (\normeda1, \normeda2);
\draw [->, magenta, thick] (0, 0) --node[left]{$c_2:=a_2 - \inner{a_2}{b_1}b_1$} (\c1, \c2);
\draw [->, teal, very thick] (0, 0) --node[right]{$\frac{c_2}{\norm{c_2}}=:b_2$} (\b3, \b4);
\draw [->, violet, very thick] (0, 0) --node[below right]{$\frac{a_1}{\norm{a_1}}\eqcolon b_1$} (\normeda1, \normeda2);
\draw [->, magenta, thick] (0, 0) --node[left]{$c_2\coloneq a_2 - \inner{a_2}{b_1}b_1$} (\c1, \c2);
\draw [->, teal, very thick] (0, 0) --node[right]{$\frac{c_2}{\norm{c_2}}\eqcolon b_2$} (\b3, \b4);
\end{tikzpicture}
\subsubsection{Beispiel}
@ -2891,7 +2892,7 @@ $
$[M = {v} \implies v \bot N]$
\item Für $M \subseteq V$ heißt
\[
M^\bot := \{ v\in V\colon v \bot M \} = \{ v \in V\colon \forall w \in M\colon \inner vw = 0 \}
M^\bot \coloneq \{ v\in V\colon v \bot M \} = \{ v \in V\colon \forall w \in M\colon \inner vw = 0 \}
\]
\underline{orthogonales Komplement} von M
\end{itemize}
@ -2948,17 +2949,17 @@ Wir zeigen: $U^\bot = \{0\} \implies (U^\bot)^{{}^\bot} =V\neq U$
Beweis wird hier nicht geführt.
\end{nonumbersatz}
\par
Sei $f \in V \setminus \{0\}, a := \norm f^2 = \inner ff, b = \norm f _\infty$.
Sei $p \in U\colon \norm{f-p}_\infty < \frac a {2b}$ \\ Behauptung: $\inner fp
> 0$
Sei $f \in V \setminus \{0\}, a \coloneq \norm f^2 = \inner ff, b = \norm f
_\infty$. Sei $p \in U\colon \norm{f-p}_\infty < \frac a {2b}$ \\ Behauptung:
$\inner fp > 0$
\begin{align*}
\inner fp = \int_0^1 f(t)p(t) dt & = \int_0^1 f(t)[f(t) - (f(t) - p(t))]dt \\
& = \int_0^1 f(t)f(t)dt - \int_0^1 f(t)(f(t) - p(t)) dt \\
& = a - \int_0^1 f(t)[f(t)-p(t)] dt \\
\int_0^1 \underbrace{f(t)}_{\substack{\le \norm b_\infty \\
\le b}}
\le b}}
[\underbrace{f(t) - p(t)}_{\substack{\le \norm{f - p}_\infty \\
\le \frac a {2b}}}]
\le \frac a {2b}}}]
& \le \int_0^1 b \cdot \frac a {2b}dt -
\int_0^1 \frac a2 dt = \frac a2 \\
\forall f \in V\colon \exists p \in U\colon \int_0^1 f(t)p(t)dt
@ -2984,7 +2985,7 @@ Sei $p \in U\colon \norm{f-p}_\infty < \frac a {2b}$ \\ Behauptung: $\inner fp
Beispiel: $U, V$ wie vorher. $\alpha \in \Hom_\R(U, V), \alpha(p) = p \forall p \in U$ \\
Angenommen $\exists \alpha^* \in \Hom_\R(V, U), e(t) = e^t \implies e \in V$ \\
$\alpha^* (e) = a_1 p_1 + \dots + a_m p_m$ \\
$f := e - (a_1 p_1 + \dots + a_m p_m) = e- \alpha^*(e) \neq 0$ \\
$f \coloneq e - (a_1 p_1 + \dots + a_m p_m) = e- \alpha^*(e) \neq 0$ \\
Behauptung: $f \in U^\bot (\implies f = 0$ \Lightning)
\item $i \in \{m+1, m+2, \dots \}$ %Eigentlich nicht so wirklich ein Punkt?!
\begin{align*}
@ -3005,7 +3006,7 @@ Sei $p \in U\colon \norm{f-p}_\infty < \frac a {2b}$ \\ Behauptung: $\inner fp
Mit $\{e_1, \dots, e_n\}$ Orthonormalbasis von $\underbrace{V}_{
\mathclap{\text{Existenz gegeben wegen Satz \ref{theo:3.1.17}}}}$ gilt:
\[
\alpha^*(w) := \sum_{i=1}^n \inner{w}{\alpha(e_i)} e_i
\alpha^*(w) \coloneq \sum_{i=1}^n \inner{w}{\alpha(e_i)} e_i
\]
\end{lemma}
\begin{proof}
@ -3022,8 +3023,8 @@ Sei $p \in U\colon \norm{f-p}_\infty < \frac a {2b}$ \\ Behauptung: $\inner fp
\begin{defin}
Sei $A \in \C^{m \times n}$.
\begin{align*}
& \overline{A} := (\overline{a}_{ij})_{i,j} & & \text{ zu $A$ \underline{konjugiert komplexe} Matrix} \\
& A^* = (\overline{A})^T & & \text{ zu $A$ \underline{adjungierte} Matrix}
& \overline{A} \coloneq (\overline{a}_{ij})_{i,j} & & \text{ zu $A$ \underline{konjugiert komplexe} Matrix} \\
& A^* = (\overline{A})^T & & \text{ zu $A$ \underline{adjungierte} Matrix}
\end{align*}
\end{defin}
@ -3042,7 +3043,7 @@ Sei $p \in U\colon \norm{f-p}_\infty < \frac a {2b}$ \\ Behauptung: $\inner fp
\begin{proof}
\begin{align*}
& A = {}_F M(\alpha)_E = (a_{ij})_{\substack{i=1,\dots,m \\
j=1,\dots,n}}, \,
j=1,\dots,n}}, \,
B = {}_E M(\alpha^*)_F = (b_{ij})_{\substack{i=1,\dots,n \\
j=1,\dots,m}} \\
& \alpha(e_j) = \sum_{i=1}^m a_{ij} f_i \\
@ -3067,7 +3068,7 @@ Sei $p \in U\colon \norm{f-p}_\infty < \frac a {2b}$ \\ Behauptung: $\inner fp
\item $\inner{\alpha(v)}{w} = \inner{v}{\alpha^*(w)} = \overline{\inner{\alpha^*(w)}{v}} =$ \\
$\overline{\inner{w}{(\alpha^*)^{{}^*}(v)}} = \inner{(\alpha^*)^{{}^*}(v)}{w} \; \forall v \in V, w \in W$ \\
$\implies \inner{\alpha(v) - (\alpha^*)^{{}^*}(v)}{w} = 0 \; \forall v \in V, w \in W, \;
w:= \alpha(v) - (\alpha^*)^{{}^*}(v)$ \\
w\coloneq \alpha(v) - (\alpha^*)^{{}^*}(v)$ \\
$\implies \inner{\alpha(v) - (\alpha^*)^{{}^*}(v)}{\alpha(v) - (\alpha^*)^{{}^*}(v)} = 0 \iff
\norm{\alpha(v) - (\alpha^*)^{{}^*}(v)} = 0 \implies \forall v \in V\colon \alpha(v) = (\alpha^*)^{{}^*}(v)$
\item
@ -3191,7 +3192,7 @@ Sei $p \in U\colon \norm{f-p}_\infty < \frac a {2b}$ \\ Behauptung: $\inner fp
\end{align*}
\begin{align*}
& \overset{\mathclap{\substack{\text{Induktionsvorraussetzung} \\
|}}}
|}}}
{\implies} \exists
\text{ ONB } (e_2, \dots, e_n) \text{ von $U$ aus Eigenvektoren von } \alpha \\
& \implies (e_1, \dots, e_n) \text{ ist ONB von $V$ aus Eigenvektoren von } \alpha
@ -3540,7 +3541,7 @@ Das sind genau die Längen- und Winkelerhaltenden Abbildungen.
\item $v = 0 \implies \inner{\alpha(v)}{\alpha(w)} = \inner{0}{\alpha(w)} = 0 \checkmark$
\item $w = \lambda v \implies \inner{\alpha(w)}{\alpha(v)} = \lambda \inner{\alpha(v)}{\alpha(v)}
= \lambda \norm{\alpha(v)}^2$. \\
Sei $l := \frac{v}{\norm v} \overset{\text{d)}}{\implies} \alpha(l)$ ist ONS
Sei $l \coloneq \frac{v}{\norm v} \overset{\text{d)}}{\implies} \alpha(l)$ ist ONS
$\implies \norm{\alpha(l)} = 1 \implies \norm{\alpha(v)} = \norm v$. \\
Es folgt $\inner{\alpha(v)}{\alpha(w)} = \inner vw \checkmark$.
\item $v, w$ linear unabhängig. Sei $(e_1, e_2)$ ONS mit $\linspan{\{e_1, e_2\}}
@ -3601,7 +3602,7 @@ Das sind genau die Längen- und Winkelerhaltenden Abbildungen.
\item Folgt direkt aus Satz~\ref{theo:3.3.2}:
\begin{align*}
\underset{\substack{\rotatebox{90}{=} \\
u+iv}}{\norm{v_\C}}
u+iv}}{\norm{v_\C}}
= 1 & \iff \norm u^2 + \norm v^2 = 1 \\
& \implies \norm{\alpha_\C(v_\C)} = \norm{\alpha(u)}^2 + \norm{\alpha(v)}^2 = 1
\end{align*}
@ -3613,8 +3614,8 @@ Das sind genau die Längen- und Winkelerhaltenden Abbildungen.
\begin{itemize}
\item $A \in \R^{\nxn}$ heißt \underline{orthogonal} wenn $A^{-1} = A^T$.
\item $A \in \C^{\nxn}$ heißt \underline{unitär} wenn $A^{-1} = A^* = \overline{A}^T$.
\item $O(n, \R) := \{ A \in \R^{\nxn}\colon \det(A)\neq 0 \land A^{-1} = A^T \}$
\item $U(n, \C) := \{ A \in \C^{\nxn}\colon \det(A)\neq 0 \land A^{-1} = A^* \}$
\item $O(n, \R) \coloneq \{ A \in \R^{\nxn}\colon \det(A)\neq 0 \land A^{-1} = A^T \}$
\item $U(n, \C) \coloneq \{ A \in \C^{\nxn}\colon \det(A)\neq 0 \land A^{-1} = A^* \}$
\end{itemize}
\end{defin}
@ -3808,7 +3809,7 @@ Betrag 1 $\cong$ unitär, positiv $\cong$ selbstadjungiert mit positiven Eigenwe
$\implies \gamma = \gamma' \implies \beta = \beta'$ \\
\underline{$\alpha$ nicht injektiv}:
\begin{itemize}
\item $W := \ker(\alpha)^\bot \implies \alpha|_W$ ist injektiv. \\
\item $W \coloneq \ker(\alpha)^\bot \implies \alpha|_W$ ist injektiv. \\
Sei $v, w \in W, \alpha(v) = \alpha(w) \implies \alpha(v - w) = 0 \implies v - w \in \ker(\alpha)
= W^\bot \cap W = \{0\} \implies v = w$
$\implies \alpha|_W = \beta_W \circ \gamma_W$ mit $\beta, \gamma \in \Hom(W, W); \beta_W$ unitär,
@ -3829,13 +3830,13 @@ Betrag 1 $\cong$ unitär, positiv $\cong$ selbstadjungiert mit positiven Eigenwe
& \inner{v}{\pi(w)} = \inner{\sum_{i=1}^n \lambda_i e_i}{\sum_{j=1}^k \mu_j e_j} = \sum_{i=1}^k \lambda_i
\overline{\mu_i}
\end{align*}
$\gamma := \pi^* \circ \gamma_W \circ \pi = \pi \circ \gamma_W \circ \pi$
$\gamma \coloneq \pi^* \circ \gamma_W \circ \pi = \pi \circ \gamma_W \circ \pi$
\begin{align*}
& v\in W & & \implies \gamma(v) = \gamma_W(v) \\
% Hier fehlt noch was aus der VO, nachschauen
& v \in W^\bot = \ker(\alpha) & & \implies \gamma(v) = 0
\end{align*}
$\beta := \underset{W}{\beta_W} \oplus \underset{W^\bot}{I}$ ist orthogonal/unitär. \\
$\beta \coloneq \underset{W}{\beta_W} \oplus \underset{W^\bot}{I}$ ist orthogonal/unitär. \\
$\implies \alpha = \beta \circ \gamma$
\end{itemize}
\end{proof}
@ -3967,7 +3968,7 @@ Polarzerlegung $A \in \K^{\nxn}, A^* A$ symmetrisch/hermitesch
Sei $V$ ein reeller/komplexer Vektorraum mit $\dim(V) = n, B=(b_1, \dots, b_n)$ Basis. Für $A \in K^{\nxn}$ ist
\begin{equation}
\label{eq:3.4.5.1}
\inner vw := {}_B \Phi(v)^T A {}_B \overline{\Phi(w)}
\inner vw \coloneq {}_B \Phi(v)^T A {}_B \overline{\Phi(w)}
\end{equation}
Genau dann ein Skalarprodukt, wenn $A$ symmetrisch/hermitesch und\\
$\underbrace{\text{positiv definit}}_{\substack{\forall \lambda \in \spec(A)\colon \lambda > 0 \\
@ -3999,10 +4000,10 @@ Polarzerlegung $A \in \K^{\nxn}, A^* A$ symmetrisch/hermitesch
\implies & a_{ij} = \overline{a_{ji}} \implies A \text{ hermitesch}
\end{align*}
Weiters muss $A$ positiv definit sein: Angenommen $\exists x \in \C^n \setminus \{0\}\colon x^T Ax = 0
\implies v := \sum x_i b_i$, das heißt ${}_B \Phi(v) = x$ erfüllt $\inner vv = {}_B \Phi(v)^T A {}_B
\implies v \coloneq \sum x_i b_i$, das heißt ${}_B \Phi(v) = x$ erfüllt $\inner vv = {}_B \Phi(v)^T A {}_B
\overline{\Phi(v)} = x^T A \overline x = 0$ \\
Sei $A$ hermitesch \& positiv definit. Klarerweise gilt dann für \\
$\inner uv := {}_B \Phi(u)^T A {}_B \overline{\Phi(v)}$:
$\inner uv \coloneq {}_B \Phi(u)^T A {}_B \overline{\Phi(v)}$:
\begin{align*}
\inner{u+v}{w} & = \inner uw + \inner vw \\
\inner uv & = \overline{\inner vu} \\
@ -4015,7 +4016,7 @@ Polarzerlegung $A \in \K^{\nxn}, A^* A$ symmetrisch/hermitesch
\begin{align*}
& \inner vv={}_B \Phi(v)^T A {}_B \overline{\Phi(v)} = {}_B \Phi(v)^T U^* \Sigma U {}_B \overline{\Phi(v)}
= \underset{\substack{\rotatebox{90}{$=$} \\
\sum \lambda_i \abs{ x_i }^2 > 0}}
\sum \lambda_i \abs{ x_i }^2 > 0}}
{(\overline U {}_B\Phi(v))^T \Sigma \overline{\overline U {}_B (v)}} \\
& v \neq0 \implies \overline U {}_B \Phi(v) =
\left(
@ -4030,7 +4031,7 @@ Polarzerlegung $A \in \K^{\nxn}, A^* A$ symmetrisch/hermitesch
Sei $A \in \K^{\nxn}$ eine symmetrische/hermitesche Matrix.
\begin{itemize}
\item \[
t(A) := \abs{ \{\lambda \in \spec(A)\colon \lambda > 0 \} }
t(A) \coloneq \abs{ \{\lambda \in \spec(A)\colon \lambda > 0 \} }
\]
heißt \underline{Trägheitsindex} von $A$.
\item $A, B$ heißen \underline{kongruent} wenn eine invertierbare Matrix $Q \in \K^{\nxn}$ existiert mit
@ -4072,7 +4073,7 @@ $M_B(\sigma), M_{B'}(\sigma)$ sind kongruent (\& umgekehrt)
& & & & & 1
\end{pmatrix}
$
$\implies S := PT$ ist invertierbar.
$\implies S \coloneq PT$ ist invertierbar.
\begin{align*}
S^* A S & = T \underbrace{P^* A P}_{\mathclap{\diag{\lambda_1, \dots, \lambda_n}}} T = T
\begin{pmatrix}
@ -4113,17 +4114,17 @@ $M_B(\sigma), M_{B'}(\sigma)$ sind kongruent (\& umgekehrt)
\end{pmatrix}
= G
\end{align*}
$t(A) := t, t(B) := s$ \\
$t(A) \coloneq t, t(B) \coloneq s$ \\
Ordne so, dass $\ontop{\lambda_1, \dots, \lambda_t > 0,
\lambda_{t+1}, \dots, \lambda_r < 0,
\lambda_{r+1}, \dots, \lambda_n = 0}
{\mu_1, \dots, \mu_s > 0, \mu_{s+1}, \dots, \mu_r < 0, \mu_{r+1}, \dots, \mu_n =0}$ \\
Setze $a_i := \sqrt{\abs{\lambda_i}}, b_i := \sqrt{\abs{\mu_i}}$
Setze $a_i \coloneq \sqrt{\abs{\lambda_i}}, b_i \coloneq \sqrt{\abs{\mu_i}}$
\begin{equation}
\label{eq:3.4.6.1}
x^* D x = \sum_{j=1}^t a_j^2 \abs{ x_j }^2 - \sum_{j=t+1}^r a_j^2 \abs{x_j}^2
\end{equation}
$C := P_2 Q^{-1} P_1, y := Cx$
$C \coloneq P_2 Q^{-1} P_1, y \coloneq Cx$
\begin{equation}
\label{eq:3.4.6.2}
x^* D x = x^* C^* G C x = y^* G y = \sum_{j=1}^s b_j^2 \abs{ y_j }^2 - \sum_{j=s+1}^r b_j^2
@ -4203,9 +4204,9 @@ Sei $A$ symmetrisch hermitesch, $\det(A) \neq 0 \implies \chi_A(\lambda) = a_1
\end{pmatrix}
\end{equation}
$V=\R^2$
$ q(x) := \sigma(x, x) = x_1^2 + x_1 x_2 + x_2 x_1 - 5x_2^2, \R^2 \to \R$
$ q(x) \coloneq \sigma(x, x) = x_1^2 + x_1 x_2 + x_2 x_1 - 5x_2^2, \R^2 \to \R$
\end{itemize}
Sei $ B= (b_1, \dots, b_n)$ Basis, so ist $M_B(\sigma) := (\sigma(b_i, b_j))_{i,j=1}^n$
Sei $ B= (b_1, \dots, b_n)$ Basis, so ist $M_B(\sigma) \coloneq (\sigma(b_i, b_j))_{i,j=1}^n$
\begin{lemma}
\label{theo:3.5.2}
@ -4365,7 +4366,7 @@ $ \implies q(\tilde x_1, \tilde x_2) = \lambda_1 \tilde x_1^2 + \lambda_2 \tilde
Sei $\rho\colon V \to \K$ heißt \underline{quadratische Form} wenn $\forall u, v \in V, \lambda \in \K\colon$
\begin{enumerate}[label=\alph*)]
\item $\rho(\lambda v) = \lambda^2 \rho(v)$
\item $ \sigma(u, v) := \rho(u + v) - \rho(u) - \rho (v)$ ist eine (symmetrische) Bilinearform
\item $ \sigma(u, v) \coloneq \rho(u + v) - \rho(u) - \rho (v)$ ist eine (symmetrische) Bilinearform
\end{enumerate}
\end{defin}
@ -4378,7 +4379,7 @@ $ \implies q(\tilde x_1, \tilde x_2) = \lambda_1 \tilde x_1^2 + \lambda_2 \tilde
$\rho$ quadratische Form $\implies \sigma(v, w) = \rho(u + v) - \rho(u) - \rho(v)$ ist symmetrische
Bilinearform. \\
Sei umgekehrt $\sigma$ symmetrische Bilinearform,
$\rho(v) := \underset{\mathclap{\substack{\rotatebox{90}{$\to$}\\\operatorname{char}(\K) \neq 2}}}
$\rho(v) \coloneq \underset{\mathclap{\substack{\rotatebox{90}{$\to$}\\\operatorname{char}(\K) \neq 2}}}
{\frac 12} \sigma(v, v)$.
\begin{align*}
\rho(\lambda v) = \frac 12 \sigma(\lambda v, \lambda v) & = \lambda^2 \frac 12 \sigma(v, v) =
@ -4401,7 +4402,7 @@ $ \implies q(\tilde x_1, \tilde x_2) = \lambda_1 \tilde x_1^2 + \lambda_2 \tilde
\begin{enumerate}[label=\alph*)]
\item $\rho(\lambda v) = \abs{\lambda}^2 \rho(v)$
\item $\rho(u+v) + \rho(u -v) = 2(\rho(u) + \rho(v))$
\item $\sigma(u,v) := \frac 12 (\rho(u+v) + i\rho(u +iv) - (1+i)(\rho(u) + \rho(v)))$ ist hermitesche
\item $\sigma(u,v) \coloneq \frac 12 (\rho(u+v) + i\rho(u +iv) - (1+i)(\rho(u) + \rho(v)))$ ist hermitesche
Sesquilinearform.
\end{enumerate}
\end{defin}
@ -4411,7 +4412,7 @@ $ \implies q(\tilde x_1, \tilde x_2) = \lambda_1 \tilde x_1^2 + \lambda_2 \tilde
\end{lemma}
\begin{proof}
Für hermitesche Form ist durch Definition \ref{theo:3.5.6} c) eine hermitesche Sesquilinearform definiert. \\
Sei umgekehrt $\sigma$ hermitesche Sesquilinearform. Dann ist $\rho(v) := \frac12 \sigma(v, v)$ hermitesche
Sei umgekehrt $\sigma$ hermitesche Sesquilinearform. Dann ist $\rho(v) \coloneq \frac12 \sigma(v, v)$ hermitesche
Form:
\begin{enumerate}[label=\alph*)]
\item \checkmark
@ -4491,7 +4492,7 @@ zusammensetzen.
\item $A^* A \in \K^{\nxn}$ selbstadjungiert und positiv semi-definit. \\
Eigenwerte $\lambda_1, \dots, \lambda_n \in [0, \infty)$, ONB $b_1, \dots, b_n$ aus Eigenvektoren.
Sei $\lambda_1, \dots, \lambda_r \in (0, \infty), \lambda_{r+1} = \dots = \lambda_n = 0$
$s_i := \sqrt{\lambda_i}, i\in [n]$
$s_i \coloneq \sqrt{\lambda_i}, i\in [n]$
\item Es gilt, dass $\overbrace{\frac 1{s_1} A b_1}^{b_1'}, \dots, \overbrace{\frac
1{s_r} A b_r}^{b_r'}$ Orthonormalsystem in $\K^m$ ist.
\begin{align*}
@ -4543,23 +4544,23 @@ $\implies \ker(\alpha) = \linspan{ b_{r+1}, \dots, b_n }_V, \im(\alpha) = \linsp
\sum_{i=1}^r s_i \lambda_i b_i' \\
\left(
\begin{smallmatrix}
x_1 \\ \vdots \\ x_n
\end{smallmatrix}
x_1 \\ \vdots \\ x_n
\end{smallmatrix}
\right) & \mapsto
\left(
\begin{smallmatrix}
x_1 \\ \vdots \\ x_r
\end{smallmatrix}
x_1 \\ \vdots \\ x_r
\end{smallmatrix}
\right) & & \mapsto
\left(
\begin{smallmatrix}
s_1 x_1 \\ \vdots \\ s_r x_r
\end{smallmatrix}
s_1 x_1 \\ \vdots \\ s_r x_r
\end{smallmatrix}
\right) & & \mapsto
\left.\left(
\begin{smallmatrix}
s_1 x_1 \\ \vdots \\ s_r x_r \\ 0 \\ \vdots \\ 0
\end{smallmatrix}
s_1 x_1 \\ \vdots \\ s_r x_r \\ 0 \\ \vdots \\ 0
\end{smallmatrix}
\right)\right\} m \\
\sum_{i=1}^r \frac{\mu_i}{s_i} b_i & \mapsfrom
\sum_{i=1}^r \frac{1}{s_i} \mu_i b_i & & \underset{\beta^{-1}}{\mapsfrom}
@ -4658,8 +4659,8 @@ Wir haben eine echte Verallgemeinerung.
$\im(\alpha) \supseteq \im(\alpha \circ \alpha^\dagger) \supseteq
\im(\alpha \circ \alpha^\dagger \circ \alpha) = \im(\alpha) \implies
\im(\alpha) = \im(\alpha \circ \alpha^\dagger)$
\item $\nu := \alpha^\dagger \circ \alpha$ erfüllt $\nu \circ \nu$ und ist selbstadjungiert
für $\nu' := \alpha \circ \alpha^\dagger$ \\
\item $\nu \coloneq \alpha^\dagger \circ \alpha$ erfüllt $\nu \circ \nu$ und ist selbstadjungiert
für $\nu' \coloneq \alpha \circ \alpha^\dagger$ \\
$\implies \underbrace{\ker(\nu)}_{=\ker(\alpha)} \bot \im(\nu)$
[Sei $v\in \ker(\nu), w = \nu(v) \in \im(\nu) \implies \inner vw = \inner{\nu(v)}{w} = 0$] \\
$\implies$
@ -4798,7 +4799,7 @@ Wir haben eine echte Verallgemeinerung.
\implies \alpha(w) \in \im(\alpha)^\bot \cap \im(\alpha) \implies \alpha(w) = 0 \\
\overset{\alpha \text{ injektiv}}{\implies} w = 0 & \text{\Lightning}
\end{align*}
$\implies \beta:= (\alpha^* \circ \alpha)^{{}^{-1}} \circ \alpha^*$ ist wohldefiniert.
$\implies \beta\coloneq (\alpha^* \circ \alpha)^{{}^{-1}} \circ \alpha^*$ ist wohldefiniert.
Nun gilt:
\begin{itemize}
\item $\alpha \circ \beta \circ \alpha = \alpha \circ (\alpha^* \circ \alpha)^{{}^{-1}} \circ \alpha^* \circ
@ -4912,7 +4913,7 @@ $\norm x = 1 \implies \norm{Ax} \ge s_1$
$\norm{b_i} \implies \lambda_1, \lambda_2 = \dots = \lambda_n = 0 \implies \norm{Ab_1} = s_1 \implies b_1$
löst unser Minimierungsproblem. \\
$Q = \{(x,y) \in \R^2\colon \psi(x, y) = 0\}$ \\
$\psi(x, y):= a_1 x^2 + a_2 xy + a_3 y^2 a_4 x + a_5 y + a_6$ \\
$\psi(x, y)\coloneq a_1 x^2 + a_2 xy + a_3 y^2 a_4 x + a_5 y + a_6$ \\
Gegeben: $(x_i,y_i)^m_{i=1}$ Suche $x = (a_1, \dots, a_6)^T$ mit $\norm x = 1$ sodass
\[
\sum_{i=1}^m \left(a_1 x_i^2 + a_2 x_i y_i + a_3 y_i^2 + a_4 x_i + a_5 y_i + a_6\right)^2=\norm{Ax}^2